Gold Will Go Up

Welcome! You are encouraged to register with the site and login (for free). When you register, you support the site and your question history is saved.

The value of gold has crashed; will it ever return to its previous high within the next year? Even at around $60, the value of gold is still up over 100 percent since it matched its last all-time high in the low $30 range just two months ago. One trend is clear, at $60 or at $250, the ecosystem of watching and trading gold continues to expand. Within the next year, without a doubt, more portfolios will include gold, more companies will be created to serve people who consider trading gold, and more people will buy gold. Clearly, it can be concluded that gold will go up.

In the argument given, the two portions in boldface play which of the following roles?

Review: Gold Will Go Up


Explanation

Reading the question: we have boldface in this question, as we did in Local Bookstores. As we discussed in that question, boldface questions have a prompt that contains one or more arguments, a question stem that asks for the role of the boldfaced statements, and answer choices that are stated in general, logical terms. We'll build a filter by analyzing how the various sentences are related and then predicting a general, logical statement.

Creating a filter: The last sentence is definitely a conclusion. We look for other instances of opinion assertion. Starting with the beginning, we see that the tone is casual, but mostly we have fact and filler. The long boldfaced sentence is a prediction: "within the next year." Since it is about the future, it's an opinion, not a fact. In arguments, a phrase like, "without a doubt," tends to mean the opposite of its denotation--it's highlighting an opinion. So the first boldfaced sentence is an opinion about the future and the second one is the author's conclusion, drawn in part on the basis of that opinion. That's our prediction.

Applying the filter: Since the second part of our prediction is simpler, we eliminate answer choices based on that part first. On those grounds, choices (B), (D), and (E) are out, and (A) and (C) are left. Choice (A) matches our prediction for the first part and (C) does not. The correct answer is (A).


If you believe you have found an error in this question or explanation, please contact us and include the question title or URL in your message.